kleine Korrekturen

This commit is contained in:
wieerwill 2022-03-28 10:08:47 +02:00
parent d989cb6a53
commit 6d447e2586
2 changed files with 14 additions and 6 deletions

Binary file not shown.

View File

@ -136,7 +136,7 @@
\begin{parts}
\part Werte die Formel $\varpi_a=\lnot p \wedge \lnot\lnot p$ im Heytingschen Wahrheitswertebereich $H_{\mathbb{R}}$ aus für die $H_{\mathbb{R}}$-Belegung $B$ mit $B(p)=\mathbb{R}\backslash \{0\}$
\begin{solution}
$B_{H_{mathbb{R}}}(\lnot p \wedge \lnot\lnot p)= Inneres(\mathbb{R}/ p)\cap p= 1$
$B_{H_{mathbb{R}}}(\lnot p \wedge \lnot\lnot p)= Inneres(\mathbb{R}/ p)\cap p= Inneres(\mathbb{R}\backslash\{\mathbb{R}\backslash\{0\}\})\cap \mathbb{R}\backslash\{0\}=\{0\}\cap \mathbb{R}\backslash\{0\} = \varnothing$
\end{solution}
\part Überprüfe ob die Formel $\varphi_B=(\lnot p\rightarrow \lnot p)\rightarrow p$ eine $K_3$-Tautologie ist. Ist $\varphi_b$ eine $B_{\mathbb{R}}$ Tautologie?
@ -148,6 +148,10 @@
$\frac{1}{2}$ & $\frac{1}{2}$ & 1 & $\frac{1}{2}$ \\
1 & 0 & 1 & 1 \\
\end{tabular}
Keine $K_3$ Tautologie.
Da keine $B$ Tautologie $\rightarrow$ keine $B_R$ Tautologie
\end{solution}
\part Überprüfe ob die semantische Folgeung $\{p\rightarrow q, q\rightarrow r\}\Vdash_B r\rightarrow\lnot p$ gilt.
@ -216,7 +220,7 @@
\item für 4.: $M_3=\{r,p\}$
\item für 1.: $M_4=\varnothing$
\end{enumerate}
\item $M_4=\varnothing \Rightarrow \varphi_b$ unerfüllbar
\item $M_4=\varnothing \Rightarrow \{\lnot\varphi_b\}$ unerfüllbar $\rightarrow \varphi$ Tautologie
\end{itemize}
\end{solution}
\end{parts}
@ -290,9 +294,13 @@
\part Gebe die Regeln $(\forall-I)$, $(\exists-E)$ und $(GfG)$ inklusive Bedingung an
\begin{solution}
$\varphi[x:=t]:\frac{\forall x\varphi}{\varphi[x:=t]}$ Bedingung: über keine Variable aus $t$ wird in $\varphi$ quantifiziert
$\forall-I:\frac{\varphi}{\forall x\varphi}$ Bedingung: x nicht frei in Hypothesen
$\exists x\varphi:\frac{\varphi[x:=t]}{\exists x\varphi}$ Bedingung: über keine Variable in $t$ wird in $\varphi$ quantifiziert
$\forall-E:\frac{\forall x\varphi}{\varphi[x:=t]}$ Bedingung: über keine Variable aus $t$ wird in $\varphi$ quantifiziert
$\exists-I:\frac{\varphi[x:=t]}{\exists x\varphi}$ Bedingung: über keine Variable in $t$ wird in $\varphi$ quantifiziert
$\exists-E:\frac{\exists x\varphi\quad \sigma}{\sigma}$ Bedingung: x weder frei in Hypothesen noch in $\sigma$
$(GfG): \frac{\varphi[x:=s]\quad s=t}{\varphi[x:=t]}$ Bedingung: über keine Variable aus $s$ oder $t$ wird in $\varphi$ quantifiziert
\end{solution}